Proof: Uniformly Continuous Function on a Bounded Set is Bounded

  • Thread starter Thread starter Quantumpencil
  • Start date Start date
  • Tags Tags
    Proof
Click For Summary
A uniformly continuous function f on a bounded set E in R is proven to be bounded on E. The discussion highlights that uniform continuity ensures that for any chosen epsilon, a corresponding delta can be established, maintaining the condition |f(x) - f(y)| < epsilon for |x - y| < delta. The initial proof attempt incorrectly assumed a specific delta based on the bounds of E, which was corrected by emphasizing the need to derive delta from epsilon. The revised argument involves selecting an arbitrary point in E and demonstrating that f remains within a bounded range around that point. This confirms the theorem that uniformly continuous functions on bounded sets are indeed bounded.
Quantumpencil
Messages
96
Reaction score
0

Homework Statement


Let f be a real uniformly continuous function on the bounbed set E in R. Prove that f is bounded on E.



Homework Equations





The Attempt at a Solution



Since f is uniformly continuous, \left|f(x)-f(y)\right|&lt; \epsilon if \left|x-y\right|&lt; \delta. Since E is bounded, there exists some maximal distance between x and y; let M be the upper limit and P be the lower limit of E. Then

\left|x-y\right|&lt; \\left|M-P\right|&lt;. If M, P are not in E, then define two sequences {x_n} and {y_n} such that lim n->infinity {x_n}=M and lim n->infinity {y_n} = P.

Then the definition of uniform continuity implies that if we let \delta = abs(M-P), so that abs(x-y) < delta for all x, y then abs(f(x)-f(y))< epsilon for all f(x), f(y), with epsilon equal to lim n->infinity {f(x_n)}=M and lim n->infinity {f(y_n)}

I feel fairly solid about this, but I'm finding the difference between absolute continuity and continuity a bit confusing and would like a rigor/correctness check.

Thanks.
 
Physics news on Phys.org
Here's where your proof goes wrong:

Then the definition of uniform continuity implies that if we let LaTeX Code: \\delta = abs(M-P) , so that abs(x-y) < delta for all x, y then abs(f(x)-f(y))< epsilon for all f(x), f(y), with epsilon equal to lim n->infinity {f(x_n)}=M and lim n->infinity {f(y_n)}

You can't just pick a delta like this. You can only pick an epsilon and then prove existence of delta using the definition of uniform continuity. Here's how I would prove it:

Choose arbitrary epsilon > 0. Then there exists delta>0 such that as long as |x-y|<delta, |f(x)-f(y)|<epsilon. Choose N so large that N delta > |M-P|. Now, choose arbitrary x0 in E. Then we must have f(x0) - epsilon N < f(x) < f(x0) + epsilon N for all x in E (check this). This proves the theorem.
 
Hm, ok. So that is the same idea I think; or at least that makes complete sense and I was just being silly.

Thanks.
 
Question: A clock's minute hand has length 4 and its hour hand has length 3. What is the distance between the tips at the moment when it is increasing most rapidly?(Putnam Exam Question) Answer: Making assumption that both the hands moves at constant angular velocities, the answer is ## \sqrt{7} .## But don't you think this assumption is somewhat doubtful and wrong?

Similar threads

  • · Replies 6 ·
Replies
6
Views
2K
  • · Replies 4 ·
Replies
4
Views
960
  • · Replies 5 ·
Replies
5
Views
2K
  • · Replies 13 ·
Replies
13
Views
2K
  • · Replies 7 ·
Replies
7
Views
1K
  • · Replies 40 ·
2
Replies
40
Views
4K
Replies
22
Views
2K
  • · Replies 5 ·
Replies
5
Views
1K
Replies
2
Views
2K
  • · Replies 27 ·
Replies
27
Views
2K